Vector triangle question - please check me on this

  • Thread starter Thread starter bcjochim07
  • Start date Start date
  • Tags Tags
    Triangle Vector
Click For Summary
The triangle formed by points A(2,-7,3), B(-1,5,8), and C(4,6,-1) is determined to be acute based on the dot products of its vectors. The calculations show that the angles BAC and ACB are acute, as their dot products are positive. Although the dot product for angle ABC is negative, it represents the supplement of the angle, confirming it is also acute. The vectors were correctly arranged for the dot product calculations, supporting the conclusion that the triangle is acute. The final determination is that the triangle is indeed acute based on the analysis provided.
bcjochim07
Messages
366
Reaction score
0

Homework Statement



The vertices of a triangle are given by points A: (2,-7,3) B: (-1,5,8), & C: (4,6,-1) Is this triangle acute, obtuse, or right?

Homework Equations


dot product is positive : acute angle
dot product is negative : obtuse angle


The Attempt at a Solution


My main question is: don't the vectors have to be arranged tail to tail before you can take the dot product to determine the angle between the vectors? That seems to be the way it is defined in my book.

The three vectors that make up the triangle are
AB: [-3,12,5]
AC: [2,13,-4]
BC: [5,1,-9]

AB\cdotAC= (-3)(2) + (12)(13) + (5)(-4)= 130 > 0, so angle BAC is acute.

AC\cdotBC = (2)(5) + (13)(1) + (-4)(-9) = 59 > 0, so angle ACB is acute
(This is the same as CA\cdotCB so they are tail to tail)

BC\cdotAB = (5)(-3) + (1)(12) + (-9)(5) = -48 < 0, but this is not the angle in the triangle, according to the picture I drew. Rather, this is the supplement, so the angle ABC is also acute.

Thus, the triangle is acute. Could somebody check my work please? Thanks.
 
Physics news on Phys.org
nota bene: The ABAC, ACBC, & BCAB are supposed to be dot products.
 
Any ideas? This assignment is due tomorrow, and I'm pretty curious about whether I am doing this problem correctly or not.
 
Question: A clock's minute hand has length 4 and its hour hand has length 3. What is the distance between the tips at the moment when it is increasing most rapidly?(Putnam Exam Question) Answer: Making assumption that both the hands moves at constant angular velocities, the answer is ## \sqrt{7} .## But don't you think this assumption is somewhat doubtful and wrong?

Similar threads

Replies
3
Views
4K
  • · Replies 1 ·
Replies
1
Views
1K
  • · Replies 2 ·
Replies
2
Views
2K
  • · Replies 13 ·
Replies
13
Views
2K
Replies
12
Views
4K
  • · Replies 1 ·
Replies
1
Views
2K
  • · Replies 1 ·
Replies
1
Views
2K
  • · Replies 2 ·
Replies
2
Views
2K
  • · Replies 1 ·
Replies
1
Views
2K
  • · Replies 2 ·
Replies
2
Views
2K